Une intégrale effrayante

Bonjour,

Elle ne se trouve dans aucune table, même les tables sacrées d' Etanche, aucun forum n'en parle, aucun moteur de recherche ne retourne une réponse. C'est une intégrale effrayante : on se perd vite dans les calculs
$$
\int_0^1 \frac {\ln(x^3+1)}{x^3+1} dx.

$$ Comme d'habitude avec ce genre d'intégrale j'ai essayé de différencier $$\int _0^1\frac{\ln \left(ax^3+1\right)}{x^3+1}\:dx$$ qui donne $$\int _0^1\frac{x^3}{\left(x^3+1\right)\left(ax^3+1\right)}\:dx$$ après je me perds dans les calculs.

Une voie prometteuse de Chaurien $\frac{\ln (x^{3}+1)}{x^{3}+1}=\frac{1}{3}(\frac{1}{x+1}+\frac{-x+2}{x^{2}-x+1})(\ln (x+1)+\ln (x^{2}-x+1))$. La première des 4 étant facile, je vous fait cadeau de la deuxième $\int _0^1\frac{\ln \left(1-x+x^2\right)}{1+x}\:dx=-\frac{\text{Li}_2\left(-3\right)}{2}-\frac{\pi ^2}{9}$

Ajout La troisième ressemble à cella https://artofproblemsolving.com/community/c7h2258910p17465551
Le 😄 Farceur


«1

Réponses

  • Maple répond quelque chose, mais même "simplifié", ce n'est pas très beau :118864
  • Bonjour,

    On ne fera pas mieux. On peut facilement trouver une série mais une expression explicite comme plus haut est préférable.
  • On a:

    \begin{align} \int_0^1 \frac{\ln(1+x^3)}{1+x^3}dx&=\ln 2\int_0^1 \frac{1}{1+x^3}dx+\frac{1}{2}\int_0^1 \frac{\ln(1-x+x^2)}{x(1-x^3)}dx+\int_0^1 \frac{\big(x\ln 2-\ln(1+x)\big)}{x(1-x^3)}dx+\\&\int_0^1 \frac{\frac{2\pi x}{\sqrt{3}}-\frac{\pi}{\sqrt{3}}+2\sqrt{3}\arctan\left(\frac{1-2x}{\sqrt{3}}\right)}{2x(1-x^3)}dx\end{align}


    La dernière intégrale risque d'être coton à calculer. :-D


    PS:
    Si vous avez des doutes sur cette égalité, voici sa "vérification" numérique avec PARI GP:
    ? A1=intnum(x=0,1,log(2)/(1+x^3));
    A2=intnum(x=0,1,log(1-x+x^2)/(x*(1-x^3)*2));
    A3=intnum(x=0,1,(x*log(2)-log(1+x))/(x*(1-x^3)));
    A4=intnum(x=0,1,(2*Pi*x/sqrt(3)-Pi/sqrt(3)+2*sqrt(3)*atan((1-2*x)/sqrt(3)))/(2*x*(1-x^3)));
    print("A1+A2+A3+A4=",A1+A2+A3+A4);intnum(x=0,1,log(1+x^3)/(1+x^3))
    
    A1+A2+A3+A4=0.1339739227498131000256196556
    %1 = 0.1339739227498131000256196569
    

    PS2:
    Une probable intégration par parties devrait améliorer le cas de la dernière intégrale.
    (Je n'ai pas fait les calculs)
  • Je me détourne avec effroi et horreur de cette plaie lamentable des fonctions $\mathrm{Li}, \mathrm{Si} $ qui seraient, soi-disant, utiles pour expliciter les intégrales.
  • On n'est pas obligé de les expliciter mais on aime bien pouvoir exprimer une intégrale à l'aide de valeurs prises par des fonctions qui appartiennent à une liste convenue.
  • Guego est ce que ça donne aussi une horreur pour $$\int _0^1\frac{\ln \left(x^3+1\right)}{x+1}\:dx$$

    FDP je commence à douter qu'on puisse trouver une expression simplifiée
    Le 😄 Farceur


  • C'est mieux :118948
  • Aha! moi je trouve mieux sauf erreur
    $\int _0^1\frac{\ln \left(1+x^3\right)}{1+x}\:dx=-\frac{\text{Li}_2\left(-3\right)}{2}-\frac{\pi ^2}{9}+\frac{1}{2}\ln ^2\left(2\right)$
    Ca colle numériquement? si oui, il y a un espoir pour simplifier l’intégrale effrayante
    Le 😄 Farceur


  • Je ne crois pas qu'il y ait d'identité qui permette de simplifier des expression comme $\displaystyle \text{Li_}2(a)+\text{Li_}2(\overline{a})$
    Mais néanmoins, on a $\displaystyle \text{Li}_2(z)+\text{Li}_2(-z)=\dfrac{1}{2}\text{Li}_2(z^2)$

    Il faudra sûrement faire intervenir des valeurs de la fonction digamma (ou de ses dérivées) si on veut une expression plus "sexy". Tout du moins, c'est l'impression que j'ai.

    PS:
    On peut donc obtenir, si je vois bien, moins de termes dans l'expression donnée par Guego.
  • FDP en tout cas ca colle bien avec la formule sexy https://www.wolframalpha.com/input/?i=-{dilog(-3)}/2-\frac{\pi+^2}{9}+\frac{1}{2}\ln+^2\left(2\right)

    je vais ouvrir un nouveau fil pour cette intégrale
    Le 😄 Farceur


  • Cette formule est compacte mais c'est plus joli des dilogarithmes avec des arguments compris entre -1 et 1 (pour ces valeurs la définition sous forme de série s'applique). B-)-
  • Un petit coup de lindep et avec un peu de baraka, on peut conjecturer que:

    $\displaystyle \int_0^1 \dfrac{\ln(1+x^3)}{1+x}dx=\frac{2}{3}\text{Li}_2\left(-\frac{1}{3}\right)-\frac{1}{3}\text{Li}_2\left(\frac{1}{3}\right)+\dfrac{1}{2}\ln^2 2+\dfrac{1}{6}\ln^2 3$

    PS:
    On peut simplifier cette expression. Lindep indique que tous les nombres (qui ne sont pas des rationnels a priori) dans le second membre ne sont pas rationnellement indépendants.
  • On peut conjecturer que:

    $\displaystyle \int_0^1 \dfrac{\ln(1+x^3)}{1+x}dx=\text{Li}_2\left(\frac{1}{3}\right)+\dfrac{1}{2}\ln^2 2+\dfrac{1}{2}\ln^2 3-\frac{1}{9}\pi^2$
  • FDP J'ai essayé à l'instant ton moteur de recherche magique (c’était plus fort que moi ) et ça donne des liens
    Le 😄 Farceur


  • Il y a aussi une jolie formule pour $\displaystyle \int_0^1 \dfrac{\ln(1+x^3)}{1+x^2}dx$ https://math.stackexchange.com/questions/3247341/evaluate-int-0-infty-frac-ln1x31x2dx

    La question

    Qui sera le premier pour donner une jolie formule pour notre intégrale effrayante
    Le 😄 Farceur


  • Il y a une relation étroite entre $\displaystyle A=\int_0^1 \dfrac{\ln(1+x^3)}{1+x}dx$ et $\displaystyle B=\int_0^1 \dfrac{\ln(1-x+x^2)}{1+x}dx$

    En effet,

    \begin{align}A&\overset{y=\frac{1-x}{1+x}}=\int_0^1 \frac{\ln\left(\frac{2(1+3y^2)}{(1+y)^3}\right)}{1+y}dy\\
    &=\ln^2 2-\frac{3}{2}\ln^2 2+\int_0^1 \frac{\ln(1+3y^2)}{1+y}dy\\
    &\overset{x=\frac{1-y}{1+y}}=-\frac{1}{2}\ln^2 2+\int_0^1 \frac{\ln\left(\frac{4(1-x+x^2)}{(1+x)^2}\right)}{1+x}dx\\
    &=-\frac{1}{2}\ln^2 2+2\ln^2 2-\ln^2 2+\int_0^1 \frac{\ln\left(1-x+x^2\right)}{1+x}dx\\
    &=\frac{1}{2}\ln^2 2+B
    \end{align}

    C'est trop compliqué on peut faire nettement plus simple :-D

    $(1-x+x^2)(1+x)=1+x^3$ :-D

    Edit: corrigé. Merci Calli.
  • très marrant :-D
    Le 😄 Farceur


  • Bonjour,
    FDP je crois que t'as écrit à plusieurs endroits 1-x²+x² au lieu de 1-x+x².
  • Ouais, mais je pense que ce changement de variable pourrait être intéressant pour notre affaire en cours.
  • On peut aussi jouer au jeu des devinettes.

    Qu'est-ce qu'il faut ajouter à notre "base" pour qu'on arrive à trouver une combinaison linéaire rationnelle qui s'annule. B-)-
  • Je vois ce que tu veux dire par ton avant message $\int_0^1 \frac{\ln(1+3y^2)}{1+y}dy $
    se calcule par dérivation de $I\left(a\right)=\int _0^1\frac{\ln \left(ax^2+1\right)}{x+1}\:dx$
    Le 😄 Farceur


  • Quand on fait le changement de variable $y=\dfrac{1-x}{1+x}$ dans l'intégrale du début on obtient:

    $\displaystyle \int_0^1 \frac{\left( y+1\right) \log{\left( \frac{2 \left( 3 {{y}^{2}}+1\right) }{{{\left( y+1\right) }^{3}}}\right) }}{3 {{y}^{2}}+1}dy$



    PS: la formule est fausse il faut que je la corrige. :-D

    Edit: corrigé.

    PS2: En décomposant en six intégrales, il y en a trois si je vois bien qu'on peut calculer facilement.
    Après, il se peut qu'en réitérant le changement de variable déjà utilisé on parvienne à calculer une ou plusieurs des intégrales restantes.
  • L’intégrale que gebrane a posé est vraiment compliqué.
    Du coup je me demande si on peut calculer $$\int_{0}^{1} \frac{ \ln(1+x^{n}) }{1+x^{n}} dx $$ avec n entier naturel non nul.
  • On entend plus parler de YvesM: effrayé aussi ? Pourtant on a de jolies formules pour n=1 et 2
    FDP peux-tu poser cette intégrale (n=3) sur MSE pour voir . Si je pose la question, le fil sera sûrement fermé . Tu as l' avantage d'etre médaillé en or.
    Dommage! Calli n'est pas passionné (en temps libres) par les sudoku à base d'intégrales
    Le 😄 Farceur


  • La dernière formule donnée par Guego, si je ne commets pas d'erreur de lecture et de copie, ne me semble pas correcte

    J'ai trouvé le problème : ce que maple appelle dilog n'est pas $Li_2$ ! On a $dilog_{maple}(x) = Li_2(1-x)$. Maple utilise la définition alternative qu'on trouve ici : https://en.wikipedia.org/wiki/Spence's_function ("Alternatively, the dilogarithm function is sometimes defined as ...")
  • Bonjour
    je signale un développement rationnel de signe alterné pour l'intégrale proposée par etanche :
    $$
    \int_0^1\frac{\ln(1+x^n)}{1+ x^n} = \frac{H_1}{n+1} - \frac{H_2}{2n+1} + \frac{H_3}{3n+1} - \frac{H_4}{4n+1} +\ldots ,

    $$ avec $H_n = 1 + \frac{1}{2} + \frac{1}{3} + \frac{1}{4} +\cdots+ \frac{1}{n},$ nombre harmonique ou somme harmonique.

    Pour $n = 3$ on retombe sur l'intégrale initiale.
    Notre ami gebrane est peut-être moins effrayé désormais ...
    Cordialement.
  • FdP a écrit:
    On peut conjecturer que :
    $\displaystyle \int_0^1 \dfrac{\ln(1+x^3)}{1+x}dx=\text{Li}_2\left(\frac{1}{3}\right)+\dfrac{1}{2}\ln^2 2+\dfrac{1}{2}\ln^2 3-\frac{1}{9}\pi^2$

    Oui, ça colle sur au moins 100 décimales.
  • @jean lismonde c’est joli le résultat que tu as trouvé
  • J'ai pris mon courage à deux mains et j'ai repris l'expression que renvoyait maple (cf http://www.les-mathematiques.net/phorum/read.php?4,2201460,2201564#msg-2201564 ) et j'ai essayé de simplifier au maximum. J'arrive finalement à :
    \[ \displaystyle \int_0^1 \dfrac{\ln(1+x^3)}{1+x^3}dx = \dfrac{\pi^2}{18} + \dfrac{\ln^2(2)}{6} + \dfrac{\ln^2(3)}{4} + \dfrac{1}{2}Li_2\left(\dfrac{1}{3}\right) + \sqrt{3}\left(\dfrac{-\pi\ln(2)}{5} + \dfrac{4\pi \ln(3)}{15} - \dfrac{3S}{5}\right) \]
    où $S =\Im\left( Li_2\left(\dfrac{3}{4} + i\dfrac{\sqrt{3}}{4}\right)\right) = \displaystyle \sum_{n=1}^{+\infty} \dfrac{1}{n^2}\left(\dfrac{\sqrt{3}}{2}\right)^n \sin\left(n\frac{\pi}{6}\right)$.
  • Bravo Guego.
    Le 😄 Farceur


  • Merci Guego.

    On peut obtenir une expression un peu plus simple:

    $\displaystyle \int_0^1 \frac{\ln(1+x^3)}{1+x^3}dx=\frac{1}{6}\ln^2 2+\frac{1}{4}\ln^2 3-\frac{1}{18}\pi^2+\frac{1}{2}\text{Li}_2\left(\frac{1}{3}\right)+\frac{1}{6}\pi\sqrt{3}\ln 3-\frac{1}{2}\sqrt{3}\Im \left(\text{Li}_2\left(\text{e}^{i\frac{\pi}{3}}\right)\right)$

    PS:
    Vous vous demandez peut-être comment je suis parvenu à cette relation?
    Je n'ai fait aucun calcul. J'ai repris les éléments de la "base" suggérée par Guego et j'ai essayé de voir si tous ces nombres étaient bien indépendants rationnellement (c'est-à-dire qu'aucun ne peut s'exprimer comme une combinaison linéaire à coefficients rationnels des autres).
    De plus, quand figure dans la "base" une expression comme $\displaystyle\Im\left(\text{Li}_2(a)\right)$, avec $a$ un nombre complexe non réel, on peut essayer de la remplacer par $\displaystyle\Im\left(\text{Li}_2\left(\frac{a}{\left|a\right|}\right)\right)$
  • Bonjour,

    Je suis admiratif.

    Ces logiciels sont impressionnants. Bravo à ceux qui les programment et ceux qui savent s’en servir.

    (tu)
  • Dans le calcul de $\displaystyle \int_0^1 \dfrac{\ln(1+x^3)}{1+x^3}dx$ on se retrouve très certainement à devoir évaluer l'intégrale $\displaystyle \int_0^1 \dfrac{\ln(1+3x^2)}{1+3x^2}dx$ c'est elle, je pense, qui donne le "méchant" terme avec la partie imaginaire dans l'expression trouvée par Guego.
    Cette intégrale se ramène au calcul de $L=\displaystyle \int_0^{\frac{\pi}{3}} \ln(\cos x)dx$.
    Et on peut conjecturer que: $\displaystyle L=\frac{1}{2}\Im \left(\text{Li}_2\left(\text{e}^{i\frac{\pi}{3}}\right)\right)-\frac{1}{3}\pi\ln 2$

    PS:
    Apparemment, si on n'aime pas les parties imaginaires de dilogarithmes de nombres complexes on peut les remplacer par une combinaison linéaire à coefficients rationnels de valeurs réelles de la dérivée de la fonction digamma.

    PS2:
    Pour calculer $\displaystyle \int_0^1 \dfrac{\ln(1+3x^2)}{1+3x^2}dx$ on commence par faire le changement de variable $y=\sqrt{3}x$ puis le changement de variable $y=\tan z$. La fonction $x\rightarrow \ln(\cos x)$ admet un développement en série.
  • FDP, justement je cherchais une fonction f qui fait apparaître ce terme méchant pour proposer cette question dans ME sous la forme d'un calcul de $\displaystyle \int_0^1 (\dfrac{\ln(1+x^3)}{1+x^3}-f(x))dx$
    Le 😄 Farceur


  • Gebrane:
    Bonne idée.
    il faudrait lui trouver un habillage plus "sexy" à ton intégrale. B-)-

    PS:
    Attention cependant, $\displaystyle \int_0^\frac{\pi}{3} \ln(\cos(x))dx$ doit être précédée d'un coefficient $\sqrt{3}$.
  • Mais attends je pense que tu fais fausse route, l’intégrale susmentionnée n'est pas le méchant ni la brute dans cette affaire puisque
    $\displaystyle \int_0^1\frac{\ln(1+a^2x)}{1+a^2x^2}dx=\frac1{2a}\arctan a\ln(1+a^2).$

    Édit ah non je n'ai pas vu le carré.
    Le 😄 Farceur


  • FDP aide moi pour que ça devient sexy
    Le 😄 Farceur


  • Quand on relit ce fil, le boulot a déjà été fait en partie par Bibi:

    $\displaystyle \int_0^1 \frac{\ln(1+x^3)}{1+x^3}dx= \int_0^1 \frac{\left( y+1\right) \log{\left( \frac{2 \left( 3 {{y}^{2}}+1\right) }{{{\left( y+1\right) }^{3}}}\right) }}{3 {{y}^{2}}+1}dy$

    Il n'y a plus qu'à soustraire la "méchante" intégrale:

    et on va obtenir:

    $\displaystyle \int_0^1 \frac{(1+x)\ln\left(\frac{2}{(1+x)^2}\right)+x\ln(1+3x^2)}{1+3x^2}$

    Ce n'est pas encore ça. On pourrait virer le $2$, dans le calcul cela donne une intégrale simple à calculer.

    PS:

    Mieux on peut virer le $x\ln(1+3x^2)$ celui-ci donne une intégrale facile à calculer.
  • Donc on obtient:

    $\displaystyle \int_0^1 \dfrac{(1+x)\ln(1+x)}{1+3x^2}dx$

    PS:
    Je vais vérifier qu'elle s'exprime toujours sur la "base" donnée par Guego.

    PS2:
    Je n'arrive plus à l'exprimer dans cette "base". Il y a eu une simplification de trop.
  • Elle est belle et moins effrayante
    Le 😄 Farceur


  • Mais je ne sais plus lui donner une forme close, j'ai été trop vite et il y a une simplification de trop.
  • alors SOS Guego.

    PS
    Guego intervient peu au forum mais ces contributions sont de qualité et très attendues. Par contre
    gebrane intervient beaucoup mais plus que la moitié de ces interventions sont vides :-D
    Le 😄 Farceur


  • J'y réfléchirai demain à tête reposée.
  • bonne nuit, je m'alite aussi
    Le 😄 Farceur


  • J'ai fait comme FdP. J'ai cherché l'intégrale comme combinaison linéaire d'éléments de notre "base". Je trouve (enfin, quand je dis "je", c'est maple :)o ) :
    \[ \displaystyle \int_0^1 \dfrac{(1+x)\ln(1+x)}{1+3x^2}dx = \dfrac{\pi^2}{54} + \dfrac{\ln^2(2)}{6} - \dfrac{\ln^2(3)}{12} - \dfrac{1}{6} Li_2\left(\dfrac{1}{3}\right) + \dfrac{2\sqrt{3}\ln(2)\pi}{15} - \dfrac{\sqrt{3}\ln(3)\pi}{15} + \dfrac{\sqrt{3}S}{15} \]
    Toujours avec $S = \Im\bigg( Li_2\Big(\dfrac{3}{4} + i\dfrac{\sqrt{3}}{4}\Big)\bigg) = \displaystyle \sum_{n=1}^{+\infty} \dfrac{1}{n^2}\Big(\dfrac{\sqrt{3}}{2}\Big)^n \sin\Big(n\frac{\pi}{6}\Big)$
  • Et tant qu'à faire :
    \[ \int_0^1 \dfrac{\ln(1+3x^2)}{1+3x^2}dx = \sqrt{3}\left(\dfrac{4\ln(2)\pi}{45} + \dfrac{\ln(3)\pi}{15} - \dfrac{2S}{5}\right) \]
  • Merci Guego, donc c'est raté car on voulait éliminer le terme méchant $\quad S = \Im\bigg( Li_2\Big(\dfrac{3}{4} + i\dfrac{\sqrt{3}}{4}\Big)\bigg)$
    Le 😄 Farceur


  • \begin{align}\int_0^1 \frac{\ln(1+3x^2)}{1+3x^2}dx\ &\overset{y=\sqrt{3}x}=\frac{\sqrt{3}}{3}\int_0^{\sqrt{3}} \frac{\ln(1+y^2)}{1+y^2}dy\\
    &\overset{y=\tan t}=-\frac{2\sqrt{3}}{3}\int_0^{\frac{\pi}{3}}\ln\left(\cos x\right)dx\\

    \end{align} Et si on pense que la valeur que j'ai donnée plus haut est correcte alors on a :
    $
    \displaystyle \int_0^1 \frac{\ln(1+3x^2)}{1+3x^2}dx=\frac{2}{9}\pi\sqrt{3}\ln 2-\frac{1}{3}\sqrt{3}\Im \left(\text{Li}_2\big(\text{e}^{i\frac{\pi}{3}}\big)\right)$

    PS. Valeur numérique :
    https://www.wolframalpha.com/input/?i=2/9*Pi*log(2)*sqrt(3)-1/3*sqrt(3)*im(polylog(2,exp(i*Pi/3)))
    https://www.wolframalpha.com/input/?i=integrate+log(1+3*x^2)/(1+3*x^2),x=0,1
  • Maintenant si on compare les deux valeurs:

    $\displaystyle \int_0^1 \frac{\ln(1+x^3)}{1+x^3}dx=\frac{1}{6}\ln^2 2+\frac{1}{4}\ln^2 3-\frac{1}{18}\pi^2+\frac{1}{2}\text{Li}_2\left(\frac{1}{3}\right)+\frac{1}{6}\pi\sqrt{3}\ln 3-\frac{1}{2}\sqrt{3}\Im \left(\text{Li}_2\left(\text{e}^{i\frac{\pi}{3}}\right)\right)$

    $\displaystyle \int_0^1 \frac{\ln(1+3x^2)}{1+3x^2}dx=\frac{2}{9}\pi\sqrt{3}\ln 2-\frac{1}{3}\sqrt{3}\Im \left(\text{Li}_2\left(\text{e}^{i\frac{\pi}{3}}\right)\right)$

    Cela permet de comprendre pourquoi mon calcul plus haut ne répond pas à nos attentes.

    Les coefficients devant le terme $\Im \left(\text{Li}_2\left(\text{e}^{i\frac{\pi}{3}}\right)\right)$ ne sont pas les mêmes.

    Si on veut faire disparaître ce terme il faut considérer $\displaystyle \int_0^1 \frac{\ln(1+x^3)}{1+x^3}dx-\frac{3}{2}\int_0^1 \frac{\ln(1+3x^2)}{1+3x^2}dx$
Connectez-vous ou Inscrivez-vous pour répondre.